Change in eigenvalues if row and column added to highly symmetric matrix Announcing the arrival of Valued Associate #679: Cesar Manara Planned maintenance scheduled April 23, 2019 at 23:30 UTC (7:30pm US/Eastern)How to calculate eigenvalues of this matrix?similar matrices, real eigenvalues, matrix rank,Sum of eigenvalues of a symmetric matrixIrreducible non-symmetric matrix with only real eigenvaluesSkew-symmetric matrix and its eigenvaluesEigenvalues for symmetric and skew-symmetric part of a matrixCan we perform row and column operations while calculating eigenvectors?Product between a column vector and a row vectorReduction of (row rank = column rank) to symmetric matricesEffect of adding a zero row and column on the eigenvalues of a matrixWhat is the eigenvalue perturbation if a row-column pair is changed?

Lagrange four-squares theorem --- deterministic complexity

How could we fake a moon landing now?

How to write capital alpha?

Flight departed from the gate 5 min before scheduled departure time. Refund options

How many morphisms from 1 to 1+1 can there be?

How to report t statistic from R

Does the Mueller report show a conspiracy between Russia and the Trump Campaign?

A term for a woman complaining about things/begging in a cute/childish way

What does Turing mean by this statement?

What does this say in Elvish?

What to do with repeated rejections for phd position

In musical terms, what properties are varied by the human voice to produce different words / syllables?

How often does castling occur in grandmaster games?

Trademark violation for app?

Should a wizard buy fine inks every time he want to copy spells into his spellbook?

Is there any word for a place full of confusion?

Why weren't discrete x86 CPUs ever used in game hardware?

Is multiple magic items in one inherently imbalanced?

What is the difference between a "ranged attack" and a "ranged weapon attack"?

Strange behavior of Object.defineProperty() in JavaScript

How can I set the aperture on my DSLR when it's attached to a telescope instead of a lens?

Tannaka duality for semisimple groups

Dynamic filling of a region of a polar plot

Do wooden building fires get hotter than 600°C?



Change in eigenvalues if row and column added to highly symmetric matrix



Announcing the arrival of Valued Associate #679: Cesar Manara
Planned maintenance scheduled April 23, 2019 at 23:30 UTC (7:30pm US/Eastern)How to calculate eigenvalues of this matrix?similar matrices, real eigenvalues, matrix rank,Sum of eigenvalues of a symmetric matrixIrreducible non-symmetric matrix with only real eigenvaluesSkew-symmetric matrix and its eigenvaluesEigenvalues for symmetric and skew-symmetric part of a matrixCan we perform row and column operations while calculating eigenvectors?Product between a column vector and a row vectorReduction of (row rank = column rank) to symmetric matricesEffect of adding a zero row and column on the eigenvalues of a matrixWhat is the eigenvalue perturbation if a row-column pair is changed?










3












$begingroup$


I have a symmetric matrix like the following:$$beginbmatrixa&a&a&a\a&b&b&b\a&b&b&b\a&b&b&bendbmatrix$$It's a symmetric real matrix with only 3 unique eigenvalues. Given it's highly symmetric nature, I was wondering how much the eigenvalues would change if I add another row and column keeping it's symmetric property intact. Specifically adding $[a, b, b, b, b]$ as a column and a row at the end.



Is there any bound for the change in eigenvalues given these sort of highly symmetric matrices?










share|cite|improve this question











$endgroup$







  • 1




    $begingroup$
    In this case you know the eigenvalues of both matrices (i.e., the before and after matrices). So what exactly would the question be?
    $endgroup$
    – M. Vinay
    Apr 2 at 2:20











  • $begingroup$
    I was looking for a generalization of the change in eigenvalues given this situation. For any dimension.
    $endgroup$
    – Hasan Iqbal
    Apr 2 at 2:25






  • 1




    $begingroup$
    You know the eigenvalues of such a matrix for any dimension, right? Or is your question indeed what its eigenvalues would be?
    $endgroup$
    – M. Vinay
    Apr 2 at 2:32






  • 1




    $begingroup$
    Hm. I think it's easier to compute it explicitly (for such a matrix of any dimension), but let's see if we can avoid that, if you really want to…
    $endgroup$
    – M. Vinay
    Apr 2 at 2:36






  • 1




    $begingroup$
    Look up Cauchy expansion of a bordered matrix and that should help.
    $endgroup$
    – Justin Stevenson
    Apr 2 at 2:38
















3












$begingroup$


I have a symmetric matrix like the following:$$beginbmatrixa&a&a&a\a&b&b&b\a&b&b&b\a&b&b&bendbmatrix$$It's a symmetric real matrix with only 3 unique eigenvalues. Given it's highly symmetric nature, I was wondering how much the eigenvalues would change if I add another row and column keeping it's symmetric property intact. Specifically adding $[a, b, b, b, b]$ as a column and a row at the end.



Is there any bound for the change in eigenvalues given these sort of highly symmetric matrices?










share|cite|improve this question











$endgroup$







  • 1




    $begingroup$
    In this case you know the eigenvalues of both matrices (i.e., the before and after matrices). So what exactly would the question be?
    $endgroup$
    – M. Vinay
    Apr 2 at 2:20











  • $begingroup$
    I was looking for a generalization of the change in eigenvalues given this situation. For any dimension.
    $endgroup$
    – Hasan Iqbal
    Apr 2 at 2:25






  • 1




    $begingroup$
    You know the eigenvalues of such a matrix for any dimension, right? Or is your question indeed what its eigenvalues would be?
    $endgroup$
    – M. Vinay
    Apr 2 at 2:32






  • 1




    $begingroup$
    Hm. I think it's easier to compute it explicitly (for such a matrix of any dimension), but let's see if we can avoid that, if you really want to…
    $endgroup$
    – M. Vinay
    Apr 2 at 2:36






  • 1




    $begingroup$
    Look up Cauchy expansion of a bordered matrix and that should help.
    $endgroup$
    – Justin Stevenson
    Apr 2 at 2:38














3












3








3





$begingroup$


I have a symmetric matrix like the following:$$beginbmatrixa&a&a&a\a&b&b&b\a&b&b&b\a&b&b&bendbmatrix$$It's a symmetric real matrix with only 3 unique eigenvalues. Given it's highly symmetric nature, I was wondering how much the eigenvalues would change if I add another row and column keeping it's symmetric property intact. Specifically adding $[a, b, b, b, b]$ as a column and a row at the end.



Is there any bound for the change in eigenvalues given these sort of highly symmetric matrices?










share|cite|improve this question











$endgroup$




I have a symmetric matrix like the following:$$beginbmatrixa&a&a&a\a&b&b&b\a&b&b&b\a&b&b&bendbmatrix$$It's a symmetric real matrix with only 3 unique eigenvalues. Given it's highly symmetric nature, I was wondering how much the eigenvalues would change if I add another row and column keeping it's symmetric property intact. Specifically adding $[a, b, b, b, b]$ as a column and a row at the end.



Is there any bound for the change in eigenvalues given these sort of highly symmetric matrices?







linear-algebra eigenvalues-eigenvectors perturbation-theory






share|cite|improve this question















share|cite|improve this question













share|cite|improve this question




share|cite|improve this question








edited Apr 2 at 2:27









Saad

20.8k92452




20.8k92452










asked Apr 2 at 2:18









Hasan IqbalHasan Iqbal

1748




1748







  • 1




    $begingroup$
    In this case you know the eigenvalues of both matrices (i.e., the before and after matrices). So what exactly would the question be?
    $endgroup$
    – M. Vinay
    Apr 2 at 2:20











  • $begingroup$
    I was looking for a generalization of the change in eigenvalues given this situation. For any dimension.
    $endgroup$
    – Hasan Iqbal
    Apr 2 at 2:25






  • 1




    $begingroup$
    You know the eigenvalues of such a matrix for any dimension, right? Or is your question indeed what its eigenvalues would be?
    $endgroup$
    – M. Vinay
    Apr 2 at 2:32






  • 1




    $begingroup$
    Hm. I think it's easier to compute it explicitly (for such a matrix of any dimension), but let's see if we can avoid that, if you really want to…
    $endgroup$
    – M. Vinay
    Apr 2 at 2:36






  • 1




    $begingroup$
    Look up Cauchy expansion of a bordered matrix and that should help.
    $endgroup$
    – Justin Stevenson
    Apr 2 at 2:38













  • 1




    $begingroup$
    In this case you know the eigenvalues of both matrices (i.e., the before and after matrices). So what exactly would the question be?
    $endgroup$
    – M. Vinay
    Apr 2 at 2:20











  • $begingroup$
    I was looking for a generalization of the change in eigenvalues given this situation. For any dimension.
    $endgroup$
    – Hasan Iqbal
    Apr 2 at 2:25






  • 1




    $begingroup$
    You know the eigenvalues of such a matrix for any dimension, right? Or is your question indeed what its eigenvalues would be?
    $endgroup$
    – M. Vinay
    Apr 2 at 2:32






  • 1




    $begingroup$
    Hm. I think it's easier to compute it explicitly (for such a matrix of any dimension), but let's see if we can avoid that, if you really want to…
    $endgroup$
    – M. Vinay
    Apr 2 at 2:36






  • 1




    $begingroup$
    Look up Cauchy expansion of a bordered matrix and that should help.
    $endgroup$
    – Justin Stevenson
    Apr 2 at 2:38








1




1




$begingroup$
In this case you know the eigenvalues of both matrices (i.e., the before and after matrices). So what exactly would the question be?
$endgroup$
– M. Vinay
Apr 2 at 2:20





$begingroup$
In this case you know the eigenvalues of both matrices (i.e., the before and after matrices). So what exactly would the question be?
$endgroup$
– M. Vinay
Apr 2 at 2:20













$begingroup$
I was looking for a generalization of the change in eigenvalues given this situation. For any dimension.
$endgroup$
– Hasan Iqbal
Apr 2 at 2:25




$begingroup$
I was looking for a generalization of the change in eigenvalues given this situation. For any dimension.
$endgroup$
– Hasan Iqbal
Apr 2 at 2:25




1




1




$begingroup$
You know the eigenvalues of such a matrix for any dimension, right? Or is your question indeed what its eigenvalues would be?
$endgroup$
– M. Vinay
Apr 2 at 2:32




$begingroup$
You know the eigenvalues of such a matrix for any dimension, right? Or is your question indeed what its eigenvalues would be?
$endgroup$
– M. Vinay
Apr 2 at 2:32




1




1




$begingroup$
Hm. I think it's easier to compute it explicitly (for such a matrix of any dimension), but let's see if we can avoid that, if you really want to…
$endgroup$
– M. Vinay
Apr 2 at 2:36




$begingroup$
Hm. I think it's easier to compute it explicitly (for such a matrix of any dimension), but let's see if we can avoid that, if you really want to…
$endgroup$
– M. Vinay
Apr 2 at 2:36




1




1




$begingroup$
Look up Cauchy expansion of a bordered matrix and that should help.
$endgroup$
– Justin Stevenson
Apr 2 at 2:38





$begingroup$
Look up Cauchy expansion of a bordered matrix and that should help.
$endgroup$
– Justin Stevenson
Apr 2 at 2:38











1 Answer
1






active

oldest

votes


















2












$begingroup$

Let $mathbf1$ denote that all-ones column vector of length $n$, and $I$ and $J$ the identity and all-ones matrices of order $n$ respectively. $newcommandonemathbf 1$



Theorem

Let $M$ be the $(n + 1) times (n + 1)$ matrix of the form
$beginbmatrixa & aone^T\ aone & bJendbmatrix$, where $a ne 0$ and $b$ are distinct real numbers. Then the eigenvalues of $M$ are:




  1. $0$ with multiplicity $n - 1$.

  2. The two roots of the equation $lambda^2 - (a + nb)lambda - na(a - b) = 0$, each with multiplicity $1$.

Proof. Since $M$ is symmetric and has rank $2$, i.e., nullity $n - 1$, it has $0$ as an eigenvalue with multiplicity $n - 1$.



Now, let $lambda$ be a root of
beginalign
lambda^2 - (a + nb)lambda - na(a - b) = 0 tag1labeleq:lambda
endalign

and define the vector $x = beginbmatrixlambda - nb \ a oneendbmatrix$ of length $n + 1$. Then



beginalign*
Mx & = beginbmatrix
(lambda - nb)a + a^2 one^T one\
(lambda - nb)a one + ab J one
endbmatrix\
&= beginbmatrix
lambda a + na(a - b)\
lambda a one
endbmatrix
endalign*

where the last step follows from $one^T one = n$ and $J one = n one$. Now observe that on rearranging eqrefeq:lambda, we get $lambda(lambda - nb) = lambda a + na(a - b)$, which shows that $Mx = lambda x$. Thus, $x$ is an eigenvector of $M$ corresponding to the eigenvalue $lambda$, for each root $lambda$ of eqrefeq:lambda. $quadsquare$






share|cite|improve this answer











$endgroup$








  • 1




    $begingroup$
    If $a = b$, then the matrix is $M = aJ_n + 1$, whose eigenvalues are $0$ with multiplicity $n$ and $(n + 1)a$ with multiplicity $1$. This can also be seen from the above, since the quadratic equation would then be $lambda^2 - a(n + 1)lambda = 0$. In the other special case, where $a = 0$ (but $b ne a$), we have $M = 0 oplus bJ_n$ (direct sum), whose eigenvalues are $0$ with multiplicity $n$ and $nb$ with multiplicity $1$. This also follows from the quadratic, which becomes $lambda^2 - nb lambda = 0$.
    $endgroup$
    – M. Vinay
    Apr 2 at 13:37











  • $begingroup$
    The reason for excluding these two special cases in the general statement is that the statement "$0$ is an eigenvalue with multiplicity $n - 1$" will not be true if they are included, and it would make some awkward rephrasing necessary.
    $endgroup$
    – M. Vinay
    Apr 2 at 13:40










  • $begingroup$
    Hi @M. Vinay, thank you so much for your answer. Could I please ask one more question? What would happen if the principal diagonal of this matrix is all zeros? Would this theorem still apply?
    $endgroup$
    – Hasan Iqbal
    Apr 2 at 13:59






  • 1




    $begingroup$
    This result certainly will not apply in that case. But a similar analysis could be done there as well. The matrix there could be written as $A + B$, where $A$ is the matrix consisting all the $a$-entries and zeroes; $B$ is the one consisting of all the $b$-entries and zeroes (i.e., $0 oplus bJ_n$). I think they'll have some common eigenvectors, so those eigenvalues would get added up to give the eigenvalues of $M$. But this is probably not true for all eigenvalues. Note that the matrix structure is $beginbmatrix0 & amathbf1^T\amathbf1 & b(J - I)endbmatrix$.
    $endgroup$
    – M. Vinay
    Apr 2 at 14:06







  • 1




    $begingroup$
    @HasanIqbal No, that's it. I'm just pointing out the structure of the modified matrix. Using that the eigenvalues can be obtained in a very similar way. That is, you assume that $beginbmatrixx\ yendbmatrix$ is an eigenvector, use block-matrix multiplication to get two equations, and solve (looking at some special case solutions usually helps you guess what the eigenvector should be like).
    $endgroup$
    – M. Vinay
    Apr 2 at 14:25











Your Answer








StackExchange.ready(function()
var channelOptions =
tags: "".split(" "),
id: "69"
;
initTagRenderer("".split(" "), "".split(" "), channelOptions);

StackExchange.using("externalEditor", function()
// Have to fire editor after snippets, if snippets enabled
if (StackExchange.settings.snippets.snippetsEnabled)
StackExchange.using("snippets", function()
createEditor();
);

else
createEditor();

);

function createEditor()
StackExchange.prepareEditor(
heartbeatType: 'answer',
autoActivateHeartbeat: false,
convertImagesToLinks: true,
noModals: true,
showLowRepImageUploadWarning: true,
reputationToPostImages: 10,
bindNavPrevention: true,
postfix: "",
imageUploader:
brandingHtml: "Powered by u003ca class="icon-imgur-white" href="https://imgur.com/"u003eu003c/au003e",
contentPolicyHtml: "User contributions licensed under u003ca href="https://creativecommons.org/licenses/by-sa/3.0/"u003ecc by-sa 3.0 with attribution requiredu003c/au003e u003ca href="https://stackoverflow.com/legal/content-policy"u003e(content policy)u003c/au003e",
allowUrls: true
,
noCode: true, onDemand: true,
discardSelector: ".discard-answer"
,immediatelyShowMarkdownHelp:true
);



);













draft saved

draft discarded


















StackExchange.ready(
function ()
StackExchange.openid.initPostLogin('.new-post-login', 'https%3a%2f%2fmath.stackexchange.com%2fquestions%2f3171379%2fchange-in-eigenvalues-if-row-and-column-added-to-highly-symmetric-matrix%23new-answer', 'question_page');

);

Post as a guest















Required, but never shown

























1 Answer
1






active

oldest

votes








1 Answer
1






active

oldest

votes









active

oldest

votes






active

oldest

votes









2












$begingroup$

Let $mathbf1$ denote that all-ones column vector of length $n$, and $I$ and $J$ the identity and all-ones matrices of order $n$ respectively. $newcommandonemathbf 1$



Theorem

Let $M$ be the $(n + 1) times (n + 1)$ matrix of the form
$beginbmatrixa & aone^T\ aone & bJendbmatrix$, where $a ne 0$ and $b$ are distinct real numbers. Then the eigenvalues of $M$ are:




  1. $0$ with multiplicity $n - 1$.

  2. The two roots of the equation $lambda^2 - (a + nb)lambda - na(a - b) = 0$, each with multiplicity $1$.

Proof. Since $M$ is symmetric and has rank $2$, i.e., nullity $n - 1$, it has $0$ as an eigenvalue with multiplicity $n - 1$.



Now, let $lambda$ be a root of
beginalign
lambda^2 - (a + nb)lambda - na(a - b) = 0 tag1labeleq:lambda
endalign

and define the vector $x = beginbmatrixlambda - nb \ a oneendbmatrix$ of length $n + 1$. Then



beginalign*
Mx & = beginbmatrix
(lambda - nb)a + a^2 one^T one\
(lambda - nb)a one + ab J one
endbmatrix\
&= beginbmatrix
lambda a + na(a - b)\
lambda a one
endbmatrix
endalign*

where the last step follows from $one^T one = n$ and $J one = n one$. Now observe that on rearranging eqrefeq:lambda, we get $lambda(lambda - nb) = lambda a + na(a - b)$, which shows that $Mx = lambda x$. Thus, $x$ is an eigenvector of $M$ corresponding to the eigenvalue $lambda$, for each root $lambda$ of eqrefeq:lambda. $quadsquare$






share|cite|improve this answer











$endgroup$








  • 1




    $begingroup$
    If $a = b$, then the matrix is $M = aJ_n + 1$, whose eigenvalues are $0$ with multiplicity $n$ and $(n + 1)a$ with multiplicity $1$. This can also be seen from the above, since the quadratic equation would then be $lambda^2 - a(n + 1)lambda = 0$. In the other special case, where $a = 0$ (but $b ne a$), we have $M = 0 oplus bJ_n$ (direct sum), whose eigenvalues are $0$ with multiplicity $n$ and $nb$ with multiplicity $1$. This also follows from the quadratic, which becomes $lambda^2 - nb lambda = 0$.
    $endgroup$
    – M. Vinay
    Apr 2 at 13:37











  • $begingroup$
    The reason for excluding these two special cases in the general statement is that the statement "$0$ is an eigenvalue with multiplicity $n - 1$" will not be true if they are included, and it would make some awkward rephrasing necessary.
    $endgroup$
    – M. Vinay
    Apr 2 at 13:40










  • $begingroup$
    Hi @M. Vinay, thank you so much for your answer. Could I please ask one more question? What would happen if the principal diagonal of this matrix is all zeros? Would this theorem still apply?
    $endgroup$
    – Hasan Iqbal
    Apr 2 at 13:59






  • 1




    $begingroup$
    This result certainly will not apply in that case. But a similar analysis could be done there as well. The matrix there could be written as $A + B$, where $A$ is the matrix consisting all the $a$-entries and zeroes; $B$ is the one consisting of all the $b$-entries and zeroes (i.e., $0 oplus bJ_n$). I think they'll have some common eigenvectors, so those eigenvalues would get added up to give the eigenvalues of $M$. But this is probably not true for all eigenvalues. Note that the matrix structure is $beginbmatrix0 & amathbf1^T\amathbf1 & b(J - I)endbmatrix$.
    $endgroup$
    – M. Vinay
    Apr 2 at 14:06







  • 1




    $begingroup$
    @HasanIqbal No, that's it. I'm just pointing out the structure of the modified matrix. Using that the eigenvalues can be obtained in a very similar way. That is, you assume that $beginbmatrixx\ yendbmatrix$ is an eigenvector, use block-matrix multiplication to get two equations, and solve (looking at some special case solutions usually helps you guess what the eigenvector should be like).
    $endgroup$
    – M. Vinay
    Apr 2 at 14:25















2












$begingroup$

Let $mathbf1$ denote that all-ones column vector of length $n$, and $I$ and $J$ the identity and all-ones matrices of order $n$ respectively. $newcommandonemathbf 1$



Theorem

Let $M$ be the $(n + 1) times (n + 1)$ matrix of the form
$beginbmatrixa & aone^T\ aone & bJendbmatrix$, where $a ne 0$ and $b$ are distinct real numbers. Then the eigenvalues of $M$ are:




  1. $0$ with multiplicity $n - 1$.

  2. The two roots of the equation $lambda^2 - (a + nb)lambda - na(a - b) = 0$, each with multiplicity $1$.

Proof. Since $M$ is symmetric and has rank $2$, i.e., nullity $n - 1$, it has $0$ as an eigenvalue with multiplicity $n - 1$.



Now, let $lambda$ be a root of
beginalign
lambda^2 - (a + nb)lambda - na(a - b) = 0 tag1labeleq:lambda
endalign

and define the vector $x = beginbmatrixlambda - nb \ a oneendbmatrix$ of length $n + 1$. Then



beginalign*
Mx & = beginbmatrix
(lambda - nb)a + a^2 one^T one\
(lambda - nb)a one + ab J one
endbmatrix\
&= beginbmatrix
lambda a + na(a - b)\
lambda a one
endbmatrix
endalign*

where the last step follows from $one^T one = n$ and $J one = n one$. Now observe that on rearranging eqrefeq:lambda, we get $lambda(lambda - nb) = lambda a + na(a - b)$, which shows that $Mx = lambda x$. Thus, $x$ is an eigenvector of $M$ corresponding to the eigenvalue $lambda$, for each root $lambda$ of eqrefeq:lambda. $quadsquare$






share|cite|improve this answer











$endgroup$








  • 1




    $begingroup$
    If $a = b$, then the matrix is $M = aJ_n + 1$, whose eigenvalues are $0$ with multiplicity $n$ and $(n + 1)a$ with multiplicity $1$. This can also be seen from the above, since the quadratic equation would then be $lambda^2 - a(n + 1)lambda = 0$. In the other special case, where $a = 0$ (but $b ne a$), we have $M = 0 oplus bJ_n$ (direct sum), whose eigenvalues are $0$ with multiplicity $n$ and $nb$ with multiplicity $1$. This also follows from the quadratic, which becomes $lambda^2 - nb lambda = 0$.
    $endgroup$
    – M. Vinay
    Apr 2 at 13:37











  • $begingroup$
    The reason for excluding these two special cases in the general statement is that the statement "$0$ is an eigenvalue with multiplicity $n - 1$" will not be true if they are included, and it would make some awkward rephrasing necessary.
    $endgroup$
    – M. Vinay
    Apr 2 at 13:40










  • $begingroup$
    Hi @M. Vinay, thank you so much for your answer. Could I please ask one more question? What would happen if the principal diagonal of this matrix is all zeros? Would this theorem still apply?
    $endgroup$
    – Hasan Iqbal
    Apr 2 at 13:59






  • 1




    $begingroup$
    This result certainly will not apply in that case. But a similar analysis could be done there as well. The matrix there could be written as $A + B$, where $A$ is the matrix consisting all the $a$-entries and zeroes; $B$ is the one consisting of all the $b$-entries and zeroes (i.e., $0 oplus bJ_n$). I think they'll have some common eigenvectors, so those eigenvalues would get added up to give the eigenvalues of $M$. But this is probably not true for all eigenvalues. Note that the matrix structure is $beginbmatrix0 & amathbf1^T\amathbf1 & b(J - I)endbmatrix$.
    $endgroup$
    – M. Vinay
    Apr 2 at 14:06







  • 1




    $begingroup$
    @HasanIqbal No, that's it. I'm just pointing out the structure of the modified matrix. Using that the eigenvalues can be obtained in a very similar way. That is, you assume that $beginbmatrixx\ yendbmatrix$ is an eigenvector, use block-matrix multiplication to get two equations, and solve (looking at some special case solutions usually helps you guess what the eigenvector should be like).
    $endgroup$
    – M. Vinay
    Apr 2 at 14:25













2












2








2





$begingroup$

Let $mathbf1$ denote that all-ones column vector of length $n$, and $I$ and $J$ the identity and all-ones matrices of order $n$ respectively. $newcommandonemathbf 1$



Theorem

Let $M$ be the $(n + 1) times (n + 1)$ matrix of the form
$beginbmatrixa & aone^T\ aone & bJendbmatrix$, where $a ne 0$ and $b$ are distinct real numbers. Then the eigenvalues of $M$ are:




  1. $0$ with multiplicity $n - 1$.

  2. The two roots of the equation $lambda^2 - (a + nb)lambda - na(a - b) = 0$, each with multiplicity $1$.

Proof. Since $M$ is symmetric and has rank $2$, i.e., nullity $n - 1$, it has $0$ as an eigenvalue with multiplicity $n - 1$.



Now, let $lambda$ be a root of
beginalign
lambda^2 - (a + nb)lambda - na(a - b) = 0 tag1labeleq:lambda
endalign

and define the vector $x = beginbmatrixlambda - nb \ a oneendbmatrix$ of length $n + 1$. Then



beginalign*
Mx & = beginbmatrix
(lambda - nb)a + a^2 one^T one\
(lambda - nb)a one + ab J one
endbmatrix\
&= beginbmatrix
lambda a + na(a - b)\
lambda a one
endbmatrix
endalign*

where the last step follows from $one^T one = n$ and $J one = n one$. Now observe that on rearranging eqrefeq:lambda, we get $lambda(lambda - nb) = lambda a + na(a - b)$, which shows that $Mx = lambda x$. Thus, $x$ is an eigenvector of $M$ corresponding to the eigenvalue $lambda$, for each root $lambda$ of eqrefeq:lambda. $quadsquare$






share|cite|improve this answer











$endgroup$



Let $mathbf1$ denote that all-ones column vector of length $n$, and $I$ and $J$ the identity and all-ones matrices of order $n$ respectively. $newcommandonemathbf 1$



Theorem

Let $M$ be the $(n + 1) times (n + 1)$ matrix of the form
$beginbmatrixa & aone^T\ aone & bJendbmatrix$, where $a ne 0$ and $b$ are distinct real numbers. Then the eigenvalues of $M$ are:




  1. $0$ with multiplicity $n - 1$.

  2. The two roots of the equation $lambda^2 - (a + nb)lambda - na(a - b) = 0$, each with multiplicity $1$.

Proof. Since $M$ is symmetric and has rank $2$, i.e., nullity $n - 1$, it has $0$ as an eigenvalue with multiplicity $n - 1$.



Now, let $lambda$ be a root of
beginalign
lambda^2 - (a + nb)lambda - na(a - b) = 0 tag1labeleq:lambda
endalign

and define the vector $x = beginbmatrixlambda - nb \ a oneendbmatrix$ of length $n + 1$. Then



beginalign*
Mx & = beginbmatrix
(lambda - nb)a + a^2 one^T one\
(lambda - nb)a one + ab J one
endbmatrix\
&= beginbmatrix
lambda a + na(a - b)\
lambda a one
endbmatrix
endalign*

where the last step follows from $one^T one = n$ and $J one = n one$. Now observe that on rearranging eqrefeq:lambda, we get $lambda(lambda - nb) = lambda a + na(a - b)$, which shows that $Mx = lambda x$. Thus, $x$ is an eigenvector of $M$ corresponding to the eigenvalue $lambda$, for each root $lambda$ of eqrefeq:lambda. $quadsquare$







share|cite|improve this answer














share|cite|improve this answer



share|cite|improve this answer








edited Apr 2 at 13:32

























answered Apr 2 at 13:24









M. VinayM. Vinay

7,35822136




7,35822136







  • 1




    $begingroup$
    If $a = b$, then the matrix is $M = aJ_n + 1$, whose eigenvalues are $0$ with multiplicity $n$ and $(n + 1)a$ with multiplicity $1$. This can also be seen from the above, since the quadratic equation would then be $lambda^2 - a(n + 1)lambda = 0$. In the other special case, where $a = 0$ (but $b ne a$), we have $M = 0 oplus bJ_n$ (direct sum), whose eigenvalues are $0$ with multiplicity $n$ and $nb$ with multiplicity $1$. This also follows from the quadratic, which becomes $lambda^2 - nb lambda = 0$.
    $endgroup$
    – M. Vinay
    Apr 2 at 13:37











  • $begingroup$
    The reason for excluding these two special cases in the general statement is that the statement "$0$ is an eigenvalue with multiplicity $n - 1$" will not be true if they are included, and it would make some awkward rephrasing necessary.
    $endgroup$
    – M. Vinay
    Apr 2 at 13:40










  • $begingroup$
    Hi @M. Vinay, thank you so much for your answer. Could I please ask one more question? What would happen if the principal diagonal of this matrix is all zeros? Would this theorem still apply?
    $endgroup$
    – Hasan Iqbal
    Apr 2 at 13:59






  • 1




    $begingroup$
    This result certainly will not apply in that case. But a similar analysis could be done there as well. The matrix there could be written as $A + B$, where $A$ is the matrix consisting all the $a$-entries and zeroes; $B$ is the one consisting of all the $b$-entries and zeroes (i.e., $0 oplus bJ_n$). I think they'll have some common eigenvectors, so those eigenvalues would get added up to give the eigenvalues of $M$. But this is probably not true for all eigenvalues. Note that the matrix structure is $beginbmatrix0 & amathbf1^T\amathbf1 & b(J - I)endbmatrix$.
    $endgroup$
    – M. Vinay
    Apr 2 at 14:06







  • 1




    $begingroup$
    @HasanIqbal No, that's it. I'm just pointing out the structure of the modified matrix. Using that the eigenvalues can be obtained in a very similar way. That is, you assume that $beginbmatrixx\ yendbmatrix$ is an eigenvector, use block-matrix multiplication to get two equations, and solve (looking at some special case solutions usually helps you guess what the eigenvector should be like).
    $endgroup$
    – M. Vinay
    Apr 2 at 14:25












  • 1




    $begingroup$
    If $a = b$, then the matrix is $M = aJ_n + 1$, whose eigenvalues are $0$ with multiplicity $n$ and $(n + 1)a$ with multiplicity $1$. This can also be seen from the above, since the quadratic equation would then be $lambda^2 - a(n + 1)lambda = 0$. In the other special case, where $a = 0$ (but $b ne a$), we have $M = 0 oplus bJ_n$ (direct sum), whose eigenvalues are $0$ with multiplicity $n$ and $nb$ with multiplicity $1$. This also follows from the quadratic, which becomes $lambda^2 - nb lambda = 0$.
    $endgroup$
    – M. Vinay
    Apr 2 at 13:37











  • $begingroup$
    The reason for excluding these two special cases in the general statement is that the statement "$0$ is an eigenvalue with multiplicity $n - 1$" will not be true if they are included, and it would make some awkward rephrasing necessary.
    $endgroup$
    – M. Vinay
    Apr 2 at 13:40










  • $begingroup$
    Hi @M. Vinay, thank you so much for your answer. Could I please ask one more question? What would happen if the principal diagonal of this matrix is all zeros? Would this theorem still apply?
    $endgroup$
    – Hasan Iqbal
    Apr 2 at 13:59






  • 1




    $begingroup$
    This result certainly will not apply in that case. But a similar analysis could be done there as well. The matrix there could be written as $A + B$, where $A$ is the matrix consisting all the $a$-entries and zeroes; $B$ is the one consisting of all the $b$-entries and zeroes (i.e., $0 oplus bJ_n$). I think they'll have some common eigenvectors, so those eigenvalues would get added up to give the eigenvalues of $M$. But this is probably not true for all eigenvalues. Note that the matrix structure is $beginbmatrix0 & amathbf1^T\amathbf1 & b(J - I)endbmatrix$.
    $endgroup$
    – M. Vinay
    Apr 2 at 14:06







  • 1




    $begingroup$
    @HasanIqbal No, that's it. I'm just pointing out the structure of the modified matrix. Using that the eigenvalues can be obtained in a very similar way. That is, you assume that $beginbmatrixx\ yendbmatrix$ is an eigenvector, use block-matrix multiplication to get two equations, and solve (looking at some special case solutions usually helps you guess what the eigenvector should be like).
    $endgroup$
    – M. Vinay
    Apr 2 at 14:25







1




1




$begingroup$
If $a = b$, then the matrix is $M = aJ_n + 1$, whose eigenvalues are $0$ with multiplicity $n$ and $(n + 1)a$ with multiplicity $1$. This can also be seen from the above, since the quadratic equation would then be $lambda^2 - a(n + 1)lambda = 0$. In the other special case, where $a = 0$ (but $b ne a$), we have $M = 0 oplus bJ_n$ (direct sum), whose eigenvalues are $0$ with multiplicity $n$ and $nb$ with multiplicity $1$. This also follows from the quadratic, which becomes $lambda^2 - nb lambda = 0$.
$endgroup$
– M. Vinay
Apr 2 at 13:37





$begingroup$
If $a = b$, then the matrix is $M = aJ_n + 1$, whose eigenvalues are $0$ with multiplicity $n$ and $(n + 1)a$ with multiplicity $1$. This can also be seen from the above, since the quadratic equation would then be $lambda^2 - a(n + 1)lambda = 0$. In the other special case, where $a = 0$ (but $b ne a$), we have $M = 0 oplus bJ_n$ (direct sum), whose eigenvalues are $0$ with multiplicity $n$ and $nb$ with multiplicity $1$. This also follows from the quadratic, which becomes $lambda^2 - nb lambda = 0$.
$endgroup$
– M. Vinay
Apr 2 at 13:37













$begingroup$
The reason for excluding these two special cases in the general statement is that the statement "$0$ is an eigenvalue with multiplicity $n - 1$" will not be true if they are included, and it would make some awkward rephrasing necessary.
$endgroup$
– M. Vinay
Apr 2 at 13:40




$begingroup$
The reason for excluding these two special cases in the general statement is that the statement "$0$ is an eigenvalue with multiplicity $n - 1$" will not be true if they are included, and it would make some awkward rephrasing necessary.
$endgroup$
– M. Vinay
Apr 2 at 13:40












$begingroup$
Hi @M. Vinay, thank you so much for your answer. Could I please ask one more question? What would happen if the principal diagonal of this matrix is all zeros? Would this theorem still apply?
$endgroup$
– Hasan Iqbal
Apr 2 at 13:59




$begingroup$
Hi @M. Vinay, thank you so much for your answer. Could I please ask one more question? What would happen if the principal diagonal of this matrix is all zeros? Would this theorem still apply?
$endgroup$
– Hasan Iqbal
Apr 2 at 13:59




1




1




$begingroup$
This result certainly will not apply in that case. But a similar analysis could be done there as well. The matrix there could be written as $A + B$, where $A$ is the matrix consisting all the $a$-entries and zeroes; $B$ is the one consisting of all the $b$-entries and zeroes (i.e., $0 oplus bJ_n$). I think they'll have some common eigenvectors, so those eigenvalues would get added up to give the eigenvalues of $M$. But this is probably not true for all eigenvalues. Note that the matrix structure is $beginbmatrix0 & amathbf1^T\amathbf1 & b(J - I)endbmatrix$.
$endgroup$
– M. Vinay
Apr 2 at 14:06





$begingroup$
This result certainly will not apply in that case. But a similar analysis could be done there as well. The matrix there could be written as $A + B$, where $A$ is the matrix consisting all the $a$-entries and zeroes; $B$ is the one consisting of all the $b$-entries and zeroes (i.e., $0 oplus bJ_n$). I think they'll have some common eigenvectors, so those eigenvalues would get added up to give the eigenvalues of $M$. But this is probably not true for all eigenvalues. Note that the matrix structure is $beginbmatrix0 & amathbf1^T\amathbf1 & b(J - I)endbmatrix$.
$endgroup$
– M. Vinay
Apr 2 at 14:06





1




1




$begingroup$
@HasanIqbal No, that's it. I'm just pointing out the structure of the modified matrix. Using that the eigenvalues can be obtained in a very similar way. That is, you assume that $beginbmatrixx\ yendbmatrix$ is an eigenvector, use block-matrix multiplication to get two equations, and solve (looking at some special case solutions usually helps you guess what the eigenvector should be like).
$endgroup$
– M. Vinay
Apr 2 at 14:25




$begingroup$
@HasanIqbal No, that's it. I'm just pointing out the structure of the modified matrix. Using that the eigenvalues can be obtained in a very similar way. That is, you assume that $beginbmatrixx\ yendbmatrix$ is an eigenvector, use block-matrix multiplication to get two equations, and solve (looking at some special case solutions usually helps you guess what the eigenvector should be like).
$endgroup$
– M. Vinay
Apr 2 at 14:25

















draft saved

draft discarded
















































Thanks for contributing an answer to Mathematics Stack Exchange!


  • Please be sure to answer the question. Provide details and share your research!

But avoid


  • Asking for help, clarification, or responding to other answers.

  • Making statements based on opinion; back them up with references or personal experience.

Use MathJax to format equations. MathJax reference.


To learn more, see our tips on writing great answers.




draft saved


draft discarded














StackExchange.ready(
function ()
StackExchange.openid.initPostLogin('.new-post-login', 'https%3a%2f%2fmath.stackexchange.com%2fquestions%2f3171379%2fchange-in-eigenvalues-if-row-and-column-added-to-highly-symmetric-matrix%23new-answer', 'question_page');

);

Post as a guest















Required, but never shown





















































Required, but never shown














Required, but never shown












Required, but never shown







Required, but never shown

































Required, but never shown














Required, but never shown












Required, but never shown







Required, but never shown







Popular posts from this blog

Triangular numbers and gcdProving sum of a set is $0 pmod n$ if $n$ is odd, or $fracn2 pmod n$ if $n$ is even?Is greatest common divisor of two numbers really their smallest linear combination?GCD, LCM RelationshipProve a set of nonnegative integers with greatest common divisor 1 and closed under addition has all but finite many nonnegative integers.all pairs of a and b in an equation containing gcdTriangular Numbers Modulo $k$ - Hit All Values?Understanding the Existence and Uniqueness of the GCDGCD and LCM with logical symbolsThe greatest common divisor of two positive integers less than 100 is equal to 3. Their least common multiple is twelve times one of the integers.Suppose that for all integers $x$, $x|a$ and $x|b$ if and only if $x|c$. Then $c = gcd(a,b)$Which is the gcd of 2 numbers which are multiplied and the result is 600000?

Ingelân Ynhâld Etymology | Geografy | Skiednis | Polityk en bestjoer | Ekonomy | Demografy | Kultuer | Klimaat | Sjoch ek | Keppelings om utens | Boarnen, noaten en referinsjes Navigaasjemenuwww.gov.ukOffisjele webside fan it regear fan it Feriene KeninkrykOffisjele webside fan it Britske FerkearsburoNederlânsktalige ynformaasje fan it Britske FerkearsburoOffisjele webside fan English Heritage, de organisaasje dy't him ynset foar it behâld fan it Ingelske kultuergoedYnwennertallen fan alle Britske stêden út 'e folkstelling fan 2011Notes en References, op dizze sideEngland

Հադիս Բովանդակություն Անվանում և նշանակություն | Դասակարգում | Աղբյուրներ | Նավարկման ցանկ